This is a Necessary Assumption question.

The stimulus says that a new screening test detects certain polyps at such an early stage that it is generally unclear whether the polyps are malignant. But the risk that a polyp might be malignant leads doctors, in most cases, to have such polyps surgically removed, which is a dangerous process. Yet some of those polyps turn out not to be malignant. All of this is the premise and we have the conclusion in the following sentence. Thus, the new screening test can prompt dangerous operations that are not actually medically necessary.

This is a classic example of a bridging Necessary Assumption question. The first part of the conclusion is fine. Because the new screening test is what leads doctors to have polyps surgically removed, “prompting” is weak enough of a claim to be supported by the premise. But look at the part of the conclusion that says these operations are “not medically necessary.” What do we know about what is or is not medically necessary? This is the bit that needs to be bridged. The starting point of the bridge is the fact that some polyps turn out not to be malignant, and the end of the bridge is that these had better not be medically necessary.

Correct Answer Choice (B) acts as this bridge. It says surgical removal of nonmalignant polyps detected by the new screening test is not always medically necessary. This has to be true. Imagine if (B) were false and surgical removal of nonmalignant polyps was always necessary. Then this argument completely fails. In other words, it must be true that removing nonmalignant polyps sometimes is not necessary. That is the bare minimum in order for the premise to have any chance of supporting the conclusion.

Answer Choice (A) says a surgical operation that is dangerous is ethically justified only for treating a medical condition that is more dangerous. You should lose interest as soon as you see “ethically justified” because the argument here is not one of ethics. It is about whether the operations are necessary or not. Even if we assume that the medical condition is not more dangerous than the operation itself, the operation would then not be ethically justified since we failed the necessary condition. But so what? Whether this is ethical is not what the conclusion is arguing for.

Answer Choice (C) says if the new screening test encourages medically unnecessary operations, then probably it either should not be used or its use should be modified. I lost interest in the first half of (C), because again, the researcher tries to argue that the test is not medically necessary. We are trying to build a bridge from her premises to that conclusion, but (C) is assuming that the conclusion is already proven and building a bridge from the conclusion to some new territory.

Answer Choice (D) says a polyp detected by the new screening test should be surgically removed if it is malignant. Of course, remove it if it is malignant. This argument is about polyps that we do not yet know to be malignant. (D) is just giving us obvious instructions.

Answer Choice (E) says the screening test is medically useful only when it detects a polyp that requires treatment. Remember how we did not care whether an operation was ethically justified in (A)? Similarly, in (E), we do not care whether a screening test is medically useful. In addition, the stimulus does not tell us which polyps require treatment. So (E) is laying out a necessary condition that itself requires a bridge (e.g., polyps at risk of malignancy require treatment) in order to even connect to the stimulus. And even if you fail this necessary condition, at best, you find out that the screening test was not medically useful.


1 comment

This is a Point at Issue/Disagreement question.

The question stem says the dialogue provides the most evidence that they disagree. Because of the words “most evidence,” I am getting the sense that the disagreement is not going to be textual, but rather inferred.

I found this question challenging, and if you did as well, I would recommend using an approach we use in comparative passages in Reading Comprehension. For these passages, I read Passage A and take a pass at the answers, then read Passage B and take another pass at the answers. I tend to not do this for Disagreement questions because they are shorter, but there is obviously a spectrum. On one end are the complicated comparative passages, and on the other end, maybe a Disagreement question where each speaker only says one sentence. Perhaps this question falls on the simpler side of the spectrum for you, but if not, I recommend siloing the information. So let's just read Paula’s claim first and take a pass at the answers.

Paula says earthlike planets seem to be the rule rather than the exception, so there are probably intelligent beings on other planets. Furthermore, we can expect our radio telescopes to detect signs of such beings. This is our main conclusion, but there is a huge jump here. Just because there are intelligent beings does not mean that we can detect them, let alone by radio telescopes.

She addresses these points in the following claims. Alien scientists would have basically the same understanding of math and physics that humans have. She does not tell us why they would, but if I accept this premise, I suppose the following subconclusion becomes a bit easier to swallow. She says the alien scientists would thus inevitably discover gravity, electromagnetism, and other fundamental physical phenomena and then develop technologies such as radio communication. Gravity and electromagnetism are consequences of our understanding of math and physics. So over time, I guess one can say alien scientists would discover these physical phenomena and develop technologies based on them. But ultimately, that aliens would develop radio communication is a subconclusion used to support the main conclusion that we can expect our radio telescopes to detect signs of intelligent beings.

Let’s now evaluate the answers. Paula has to either agree or disagree with each of these answer choices, i.e., she has to support it or anti-support it. Any answer choice that Paula has expressed no opinion on is unsupported and immediately wrong.

Answer Choice (A) says they disagree about whether it is probable that there are intelligent alien beings who have developed radio communication. Paula says there probably are intelligent alien beings, and that alien scientists would have developed radio communication. If we combine the two claims, we can say it is probable that there are intelligent alien beings who have developed radio communication. (A) is supported.

Answer Choice (B) says they disagree about whether it is probable that there are intelligent alien beings who have the same legal or political systems that humans do. Paula did not say anything about legal or political systems, so this is unsupported. That means we can eliminate this answer.

Answer Choice (C) says they disagree about whether our technology is influenced by our cognitive makeup. You might be tempted to say Paula disagrees with this. Maybe she believes alien scientists would have the same understanding of math and physics as us because there is something objective and absolute about them that does not depend on unique physiology or cognitive makeup. And our technology depends on math and physics, so it is not influenced by physiology or cognitive makeup.

But remember that Paula did not explain why alien scientists have the same understanding of math and physics as humans. She could believe this is so because aliens have the same physiology or cognitive makeup as humans. You cannot decide between those two possibilities because her statements are consistent with either reading. (C) is unsupported and therefore we can eliminate it.

Answer Choice (D) says they disagree about whether there is likely to be intelligent life on other planets. Paula explicitly says this, so (D) is supported.

Answer Choice (E) says they disagree about whether scientists should spend more time and money on the search for intelligent extraterrestrial life. Paula does not talk about whether the amount of investment into the search for extraterrestrial life is enough. So (E) is unsupported and eliminated.

Let’s now move onto Ashley’s claims. Under timed conditions, you would only evaluate (A) and (D) because you have ruled out (B), (C), and (E) based on Paula’s claims. But since we are reviewing, let’s look at all of them.

Ashley says Paula’s claim is like saying they, the aliens, would inevitably have the same legal or political systems that humans do. Do you detect the sarcasm here? She says our science, math, and technology are unique outgrowths of our physiology, cognitive makeup, and environment. And her conclusion is that using radio telescopes to search for intelligent life is a waste of time and money.

Notice that while Ashley is not committed to whether aliens exist, she is saying that even if they did exist, they would not have the same science, mathematics, and technology as us. Ours are unique, so this implies that aliens would have different ones, just as they would have different legal or political systems if they even had one. And this is why Ashley thinks using radio telescopes to search for intelligent life is a waste of time and money. She does not believe aliens have radio communication.

Answer Choice (A) is anti-supported. Ashley would absolutely disagree that there probably are alien beings who have radio communication.

Answer Choice (D) is unsupported. While Ashley would say radio-communicating intelligent alien beings probably do not exist, everything she says is consistent with her believing that there are or are not some intelligent beings on other planets.

Answer Choice (C) is supported. Ashley believes our technology is a unique outgrowth of our cognitive makeup.

Answer Choice (B) is anti-supported. Ashley is making fun of Paula by talking about aliens who have the same legal or political systems as humans. It is strongly implied that she thinks there are not any.

Answer Choice (E) is unsupported. (E) is talking about general investment into the search for intelligent life. Perhaps Ashley thinks that we should keep searching for aliens, but in some other way.

So from Ashley's perspective, we can get rid of (D) and (E). If you combine the results of both, the only answer choice remaining is the Correct Answer Choice (A). Paula agrees with this. Ashley disagrees with this.


8 comments

This is a Strengthening question.

The psychologist's argument uses scientific reasoning, so we have a phenomenon and a hypothesis. The stimulus describes some observations, which can easily sound like an experiment. If interpreted as an experiment, it's a pretty badly designed experiment. So thinking about what controls we need to improve this "experiment," in other words, contrasting this with what an ideal experiment would look like, will help you solve this question.

The stimulus says most people's blood pressure rises when they talk, but extroverted people experience milder surges when they speak than do introverted people, for whom speaking is more stressful. So we have kind of a classic setup in the premise with two different groups of people and two different results. And the conclusion, or the hypothesis, is that this suggests that blood pressure increases result from the psychological stress of communicating rather than from the physical exertion of speech production.

This is strange. What if physical exertion is something that does affect blood pressure? It sounds reasonable to say that this might be a competing explanation that we would want to examine. If the psychologist had said that it is stress that causes blood pressure to increase and not, for example, the color of the hat someone is wearing, then sure. It makes sense that we would not examine the color of people’s hats since this is unlikely to affect blood pressure.

But the conclusion is ruling out physical exertion while the premise did not give us reasons to do so. In fact, physical exertion is not mentioned at all. And that is why this is a badly designed "experiment." It is not really even an experiment, but rather just an observation, as it does not control for physical exertion or anything else, for that matter. Imagine how weak this argument would be if introverts happened to physically exert more when they spoke than extroverts. Now not only does stress vary with blood pressure, physical exertion also varies with blood pressure. How can you say it is the stress, and not the physical exertion, that is causing the surge in blood pressure?

If you really wanted to figure out if it is psychological stress that causes increases in blood pressure, you would collect a bunch of people and randomly split them into two groups. You would not care if they were introverts or extroverts because these distinctions only mattered since they were thought to be good proxies for the speakers’ stress levels. The conclusion wasn't that introversion caused an increase in blood pressure. It was that stress caused the increase. And because the groups are totally random, people who do a lot of physical exertion and people who do not would be spread out across the two groups as well. Now you have controlled for physical exertion.

And then you intervene in one of the groups. It does not matter which one because they are equalized at the moment. You intervene by stressing out one of the groups as they speak (maybe by inducing conversation on controversial topics), and whatever differential blood pressure you observe, you can be pretty sure it was due to your psychological intervention. That is how you design an ideal experiment.

So you have two options if you want to strengthen this crappy argument. One option is to control for physical exertion: say that the amount of physical exertion is the same for both introverts and extroverts. Another is to say that physical exertion simply does not matter for blood pressure, just as the color of somebody’s hat does not matter.

Correct Answer Choice (D) strongly suggests that physical exertion doesn't matter. On first blush, (D) might seem irrelevant because it does not mention introverts, extroverts, or blood pressure. This is why I generally try not to anticipate answers in Strengthening or Weakening questions, especially in scientific reasoning. LSAT writers can always think of things I cannot think of, and I might be digging myself into a hole by trying to anticipate the answer.

(D) says deaf people experience increased blood pressure when they sign, i.e., communicate, but no change when they move their hands for any other reasons. So (D) is suggesting that it is not the physical exertion that is causing blood pressure to increase. If it were, you would expect to see an increase in both situations.

Do you see how physical exertion, while not equalized, has now at least been somewhat accounted for? (D) does not control for differences in physical exertion between the two groups (for example, it could still be the case that introverts move their hands more than do extroverts) but it does strongly suggest that even if there were differences in physical exertion, it would not be relevant, like the color of your hat.

So what is causing this blood pressure increase? In one situation, they are communicating, and in the other, they are not. So (D) is in fact also hinting that the stress attendant with communication might be causing blood pressure to increase. So (D) is great. Even with (D), you can still poke holes in the argument, but this is okay. In Strengthening questions, you just need to make the argument a little bit better.

Answer Choice (C) says introverted people who do not have chronically high blood pressure often sense the rises in blood pressure that occur when they speak in conversation. So there is a subset of introverts who do not have chronic high blood pressure, and this subset can sense their blood pressures go up. Maybe they feel flushed, dizzy, etc. This doesn't help the argument. Imagine if this group was not so sensitive and could not sense this increase. This would not change the fact that the blood pressure for this group still went up.

In addition, how does this group of introverts relate to introverts in general? And what about introverts who do have chronically high blood pressure? Are they as sensitive? What about extroverts who do or do not have chronic high blood pressure? (C) is silent on all of these questions and completely useless.

Answer Choice (E) is a little better than (C) in that at minimum, it is a comparison. (C) just told us something about a random subset of introverts, but (E) says extroverts are more likely to have chronically high blood pressure than are introverts. On top of that, extroverts are also more likely to take medication to lower their blood pressure.

So (E) is actually kind of relevant, and if anything, it is revealing another hole in the reasoning. It is telling us that extroverts are potentially carrying in this baggage that could undermine the conclusion. If extroverts are taking medication to lower their blood pressure, the observed difference in blood pressure between the two groups could just have been the result of this medication. (E) is offering a competing explanation for why the extroverts’ blood pressure did not go up as much. And I rarely say this, but if this were a Weakening question, I think (E) would actually be a great answer choice.

Answer Choice (A) is kind of interesting in its relationship to (E). (A) says medications designed to lower blood pressure do not keep people who take them from experiencing blood pressure fluctuations when speaking. We saw how (E) weakened the argument by presenting a potential alternative hypothesis. (A) is addressing this very weakness by saying that the extroverts’ medications do not prevent blood pressure surges. Maybe they are designed to just lower ambient levels of blood pressure and do not work when your blood pressure surges.

I do not think (A) is very relevant without its relation to (E), because when we consider (A) alone, we do not even know which group is taking this medication. So where (A) might be relevant is precisely when it saps energy away from (E). But notice how we are now just back to where we were in the beginning. The argument is not any stronger.

Answer Choice (B) says that in general, the lower one's typical blood pressure, the more one's blood pressure will increase under stress. Fair enough, but it seems like (B) is ignoring the argument and just reiterating the conclusion that stress makes your blood pressure go up. This is a classic wrong answer choice in Strengthening questions. It just leaves our argument as good or bad as it ever was.

First, we do not know who has the lower baseline blood pressure since the stimulus just said extroverts experience a milder surge than introverts. And even if we say that the higher surges for introverts were due to a lower baseline and milder surges for extroverts were due to a higher baseline, we are still left with the original hole in the argument. We do not know if it is the stress or the physical exertion that causes surges in blood pressure.


16 comments

This is a Flaw/Descriptive Weakening question.

The stimulus says that if the proposed air pollution measures were to be implemented, ozone levels in the city's air would be one fifth lower, i.e., 20% lower, than current levels. Since the ozone in our air is currently responsible for $5 billion in health costs (premise), we would spend about a billion dollars less on these ozone-related health costs should the proposal be adopted (conclusion).

We always have two options when approaching Flaw questions. Either we identify the flaw in advance and go hunting for it in the answer choices, or use process of elimination. If you think this argument makes sense, keep an open mind as you go through the answer choices because the correct answer will point out something that you had not considered.

Answer Choice (A) says the argument fails to consider the possibility that other types of pollution not involving ozone might rise, perhaps even producing an overall increase in health costs. Sure, maybe particulate matter pollution or carbon dioxide pollution will rise. But the argument is completely contained to ozone and does not contemplate non-ozone-related health costs. So to criticize it for failure to do that is not a criticism of the logic of the argument.

(A) might be a fair criticism if we were having a discussion about health costs in general. But when we evaluate arguments in Weakening or Flaw/Descriptive Weakening, we have to limit that evaluation to the actual premise and conclusion presented, all of which are limited to ozone here. If (A) flies, then I can also say that the biggest contributor to health costs is heart diseases, not ozone, so we should talk about heart diseases if we really want to reduce health costs.

Correct Answer Choice (B) says the argument presumes, without providing evidence, that ozone-related health costs in the city vary roughly in proportion to ozone levels. This means if you reduce ozone levels by 20%, health costs would also go down by 20%. At a minimum, you should recognize that that is an accurate description of the assumption made. The argument is in fact presuming this, so (B) passes step one of the two-step test.

Now ask yourself if it is in fact reasonable to assume this. It turns out it is not. Ozone levels could generate health costs once the level of ozone passes a certain threshold. So it could be that ozone pollution is negligible until after a critical mass of the pollution has been accumulated, after which it becomes very damaging. If that were the case, then the 20% reduction might bring ozone levels under the threshold, which would result in health benefits of $5 billion. The opposite could also be true. Ozone levels could still be above the threshold even after the reduction, in which case we might not reduce health costs at all.

I am not saying this is how ozone levels actually work, but the point is that because we do not know how they work, we cannot make the naive assumption that the relationship between ozone levels and health care costs is proportional. There are so many other non-proportional relationships. And finally, in reality, the economic concept of diminishing marginal returns cuts against the assumption of proportionality.

Answer Choice (C) says the argument provides no explicit reason for believing that the proposed air pollution measures will in fact be adopted. Like (B), (C) is descriptively accurate. We do not know if the measures will be adopted or not. However, this is not the flaw. The premise says, “if the proposed air pollution measures were to be implemented,” so it is contemplating a hypothetical world. If we adopt it, what would happen?

Answer Choice (D) says the argument attempts to support the conclusion by making an appeal to emotions. The conclusion is supported by an appeal to math, not emotions. We think we would reduce health costs by 20% because the ozone levels will go down by 20%. An argument that did appeal to emotions would say something like “we should adopt the new ozone control measures because Timmy lost his mother to ozone pollution.”

Answer Choice (E) says the argument discusses air pollution to draw attention away from more significant sources of health-related costs. An argument actually guilty of this vulnerability would establish that a more significant source of health cost was, for example, heart disease. And the author would say, have you guys considered air pollution? There is ozone, nitrogen, volatile organic compounds, etc. That is trying to draw attention away from heart disease, which does not happen here.


Comment on this

This is a Necessary Assumption question.

The stimulus says recently discovered clay tablets from southern Egypt date to between 3,300 and 3,200 B.C. Though most of the tablets translated thus far are tax records, one of them appears to contain literary writing. All of this is premise supporting the conclusion.

And the conclusion is that these tablets challenge the widely held belief among historians that the Sumerian civilization in Mesopotamia was the first to create literature. So we used to think that the Sumerians were the first, and somehow these tablets challenge that, meaning the Egyptians predate the Sumerians. If you already spotted this assumption, you can just go hunting for the correct answer. But we’ll use process of elimination here.

Answer Choice (A) says most of the recently discovered tablets that have not yet been translated contain literary writing. So in the entire set of recently discovered tablets, there is a subset that has not yet been translated. And in that subset, most contain literary writing.

We do not need this to be true. We already have one of the tablets that contain literary writing and that is enough. Sure, having more would strengthen this argument, but we are just trying to find the necessary assumption. Running the negation test also helps. Say not most but rather just a few of the tablets contain literary writing. The argument does not fall apart.

Answer Choice (B) says every civilization that has kept tax records has also kept other written records. This is also not necessary. We are only talking about two civilizations, the Egyptian and the Sumerian. Why do we care if some other civilization like the Aztecs kept other written records? What does that have to do with this argument? Egypt still predates, or does not predate, the Sumerians in creating literature.

Correct Answer Choice (C) says historians generally believe that Sumerians did not create literature earlier than 3,300 B.C. This has to be true. Imagine if historians generally believed that the Sumerians did create literature earlier than 3,300 B.C., say 4,000 B.C. That is 6,000 years ago. And now we have this Egyptian tablet from 3,300 B.C., only 5,300 years old at best. How is this supposed to challenge the belief that Sumerians were the first to create literature? The Sumerians still predate the Egyptians by 700 years. That is why (C) is absolutely necessary.

Note that this question could have been way harder. Imagine if one of the other answer choices said historians generally believe that Sumerians first created literature between 2,800 B.C. and 2,700 B.C. This would be a super attractive answer choice. However, while this would certainly help the argument by definitively showing that Egyptian literature is older by about 600 years, it is not necessary. And you can see this is not necessary by changing the dates a bit, say 2,500 and 2,400 B.C. That would also help the argument. So it is not necessary that historians have to believe in the 2,800 to 2,700 B.C. date range.

Answer Choice (D) says some historians are skeptical about the authenticity of recently discovered tablets. This is not necessary. If anything, the skepticism only hurts the credibility of the argument. Necessary Assumption is part of the superset that we call Strengthening.

Answer Choice (E) says the Sumerian civilization arose sometime between 3,300 B.C. and 3,200 B.C. This is also not necessary. What if it instead arose between 2,800 and 2,700 B.C.? Falsifying, or negating, (E) does not ruin the argument. If anything, this version of falsification actually helps the argument. Egypt would clearly predate the Sumerians in this scenario because the Egyptian clay tablet would be dated to be 500 years older than the Sumerian civilization itself.


5 comments

This is a Main Conclusion question.

This question is pretty cookie-cutter. Structurally, we are given 1) a phenomenon, 2) other people's hypothesis, and 3) an alternate hypothesis. What makes this question challenging despite this cookie-cutter structure is the presence of two really attractive answer choices.

Let's first look at the stimulus. The climatologist says the waters off the Pacific coast of North America have warmed about 4 degrees over the last 15 years. This is the phenomenon. And then she tells us other people’s hypothesis first (you can also call it an explanation or a conclusion, but I will call it a hypothesis here for simplicity). Some scientists claim that this trend of warming is a symptom of a more general global warming caused by human-generated air pollution.

And our author, predictably, says this conclusion from other scientists is far from justified. That is our author's conclusion. And why should we believe this? Because of her premise. Because it is known that there are many natural cycles of ocean temperature changes that last 60 years or more. Her premise is an alternate hypothesis for the very same phenomenon. The other scientists’ claims are far from justified because they have not considered this alternate hypothesis.

That is the stimulus, but the answer choices get kind of tricky.

Let’s start with the Correct Answer Choice (C), which says that the conclusion that the warming of the waters off the Pacific coast of North America is a symptom of a more general, global warming caused by human-generated air pollution is far from justified. This is a perfect fully fleshed out version of “this conclusion is far from justified,” where “this conclusion” refers to some scientists’ claims that the warming of waters is caused by a general global warming.

Contrast (C) with Answer Choices (B) and (D), two really attractive choices. I think both (B) and (D) would be correct in Reading Comprehension Inference questions that ask what the author would agree with. I think she would agree that the warming of the waters is not a symptom of global warming. I think she would also agree that the warming of the water may be the result of natural cycles, and this is probably even better supported since it uses softer language. But neither is the main conclusion.

Let’s look at (B) first. (B) is a cookie-cutter wrong answer choice. There are many Flaw questions that explicitly test this error made in (B).

Say a patient shows up with symptoms of high fever, and her first doctor says this is because of the common cold. And then a second doctor says, "This conclusion is far from justified." Why? Her premise to support this conclusion is some alternative hypothesis that might also explain the phenomenon, like the flu.

If this were a Flaw question, you have to be careful in how you describe what the second doctor is doing. She is questioning the first doctor's reasoning for failing to consider an alternative hypothesis. Put differently, the second doctor is merely questioning the support structure of someone else's argument. That is not the same thing as concluding that the patient does not have a cold, or that the patient indeed has the flu.

If this were an Inference from the author's perspective question, then you may well say that the second doctor thinks that the patient doesn't have a cold. But just because she revealed the weakness in someone's reasoning does not mean that she has shown that their conclusion is false. In order to do that, she still has to come up with her own argument to support what we think she believes: that the patient doesn't have a cold.

(B) is making this mistake by saying the warming of the waters off the Pacific coast is not a symptom of a more general global warming. That is not the author's conclusion, even if we think the author would agree. The author's conclusion is simply that people who claim this have not done a good job of supporting this claim.

And (D) makes a similar mistake. (D) says the warming of the waters off the coast may be the result of a natural cycle of ocean temperature changes. The climatologist would probably agree with this, but it does not matter because the question is asking us to identify the main conclusion.

Answer Choice (E) is similar. (E) says if the warming of the waters off the Pacific coast of North America is due to natural cycles of temperature change, then it is not a symptom of global warming. Like (B) and (D), I think the author would agree with (E). (E) could be some sort of an underlying principle that says you do not need to consider a hypothesis if a phenomenon can be explained by some other hypothesis. But that does not mean it is the main conclusion of the argument.

Answer Choice (A) says some scientists have found evidence that the waters off the Pacific coast have grown warmer over the past 15 years. I do not even know if this is true. The phenomenon of waters warming is declared as a fact without any evidence offered, and this is certainly not the main conclusion.


1 comment

This is a Point at Issue/Disagreement question.

The question stem says the statements commit Robin and Kendall to disagree. Because of the word commit, I am getting the sense that the disagreement is not going to be explicit, but rather strongly implied.

Robin says archeologists can study the artifacts left by ancient cultures to determine whether they were nomadic or sedentary. If the artifacts were made to last as opposed to quickly discarded, then the culture was likely sedentary. Note that this implies that if the artifacts were made to be discarded, the culture was likely nomadic. Robin lays out only two cultures, nomadic or sedentary; and only two types of artifacts, durable or non-durable. So the non-durable type is pointing to (evidence of) a nomadic culture whereas the durable type is evidence of a sedentary culture.

A note about the "arrows." The sentence is presented using "if" so you might be tempted to map this out as a conditional relationship. It wouldn't be wrong, though it would be overlooking the actual meaning of the sentence which is implicitly expressing a causal relationship. Think about why the type of artifact is evidence of the type of culture. In other words, why would finding durable artifacts point to the culture's being sedentary? The implicit reason is because certain types of culture are more likely to make certain types of artifacts. In other words, sedentary culture causes the production of durable artifacts. The logic here is causation. A tends to cause X whereas B tends to cause Y. A and B are long gone and all we have left are X and Y. So finding X or Y is the evidence of A or B. That's the underlying logic. It's not conditional. It's causal.

Kendall then says what artifacts a people make is determined largely by the materials available to them. Kendall is telling a different causal story. For example, if they were next to a forest, they probably made artifacts out of wood. If they were next to a quarry, out of stone. Because Kendall has a different causal story (hypothesis) in mind, he's thinking that these artifacts may not tell you about the type of culture they were. Rather, they might only be evidence of what materials the cultures had access to.

Okay, so this is a Disagree question. Recall your spectrum of support. On one end we have supported, on the other end anti-supported, and right in the middle is unsupported, i.e., it is neither supported nor anti-supported. We are looking for an answer choice where you can land one speaker on one end and another on the other end. And if either speaker lands in the unsupported territory in the middle, the answer choice is automatically wrong because you cannot disagree with someone who has not expressed a position.

Answer Choice (A) says they disagree over whether the distinction that Robin makes between two kinds of cultures is illicit, i.e., not allowed. Robin would, of course, disagree with this. She makes the distinction between nomadic and sedentary. Kendall has expressed absolutely no opinions on this. Kendall just says artifacts do not tell you much about this distinction. Robin is in the anti-supported territory and Kendall is in the unsupported territory.

Correct Answer Choice (B) says they disagree over whether it is reasonable to assume that a culture whose artifacts were not durable was nomadic. Robin says if the artifacts were durable, the culture was likely sedentary, so it is implied that if they were not durable, they were nomadic. So Robin agrees with (B). But Kendall disagrees. Kendall thinks that non-durable artifacts may simply be evidence of the kinds of (non-durable) materials the culture had access to, regardless of whether that culture was sedentary or nomadic. For Kendall, (B) is anti-supported.

Answer Choice (C) says they disagree over whether any evidence other than the intended durability of a culture's artifacts can establish conclusively which of the two kinds of cultures a particular culture was.

The difficulty of (C) is just in the grammar. (C) is talking about whether we can conclude a culture was nomadic or sedentary based on some feature other than the durability of artifacts. For example, maybe we can point to the establishment of cities as evidence that they were sedentary. Maybe we can point to sparse traces of a culture that span a vast territory as evidence that they were nomadic. Neither Robin nor Kendall expresses any opinion on this, so (C) sits at the unsupported territory for both.

For (C) to be correct, one of them, probably Robin, would have to say that the only thing you can look at to determine if a culture was sedentary or nomadic is the durability of their artifacts. And Kendall would say you can also look at other features like traces of cities or movement patterns.

Answer Choice (D) says they disagree over whether the distinction that Robin makes is as important as many archeologists have thought. From Robin’s perspective, it is unclear whether this is as important because we do not know exactly how important archaeologists thought this distinction was in the first place. And Kendall did not say anything at all about nomadic versus sedentary cultures. So (D), like (C), is totally unsupported for both.

For (D) to be correct, we would need someone, likely Robin, saying archeologists make this distinction because it is important, and one of the ways they study this is by looking at artifacts. And Kendall would say this distinction is not that important, and that is why most archeologists study something else.

Answer Choice (E) says they disagree over whether studying a culture's artifacts can reveal a great deal about the culture. Both Robin and Kendall would probably agree here. Robin thinks that studying artifacts can reveal whether a culture was nomadic or sedentary. Kendall thinks that studying artifacts can reveal the types of materials they had access to. So they both agree that it can reveal a great deal. They just disagree over what the revelation is because they disagree over the causes of the artifacts in the first place.


8 comments

This is a Main Conclusion question.

I think a little bit of economics knowledge is helpful for this question. The stimulus says railroads rely increasingly on automation. And since fewer railroad workers are needed (premise), operating costs have been reduced (conclusion). When an industry increasingly relies on automation, it means it needs fewer workers, right? And as a result, operating costs have been reduced. It is cheaper to operate railroads now. Let’s read on to see if this is the main conclusion or a sub-conclusion.

The stimulus then says this means that we can expect the volume of freight shipped by rail to grow, with “this” referring to the fact that it is cheaper to operate railroads. “Operating costs have been reduced” now sounds like a sub-conclusion. Why should I believe that railway shipping volume will grow? Because railway is cheaper. People are now going to use more of it. This is not an airtight argument. There are multiple assumptions at play. One is that because operating costs are reduced, price will be reduced. Another is that because price is reduced, demand will increase. Noted, but we're just looking for the main conclusion here.

Now the last sentence says that the chief competitor of railway shipping is shipping by truck, and no reduction in operating costs is predicted for the trucking industry. If we make the same two assumptions again here, that means prices for trucking are not going to change. And because trucks are railway’s competitors, that means relative to trucks, railway's price is cheaper. If we assume that relative costs affect demand, then that's also support for the conclusion that the volume of freight shipped by rail will grow. So that's the main conclusion.

If this were a Weaken or Strengthen question, then we would definitely latch onto any of those assumptions. But this is just an MC question.

Correct Answer Choice (A) is the correct paraphrasing. The volume of freight shipped by rail can be expected to increase.

Answer Choice (B) could be correct in “inference from the author's perspective” questions in Reading Comprehension that ask for the claim the author would most likely agree with. (B) says increasing reliance on automation means that fewer railroad workers are needed. While this is definitely implied in the first two sentences of the stimulus, it is not the main conclusion.

Answer Choices (C) and (E) say no reduction in operating cost is predicted for the trucking industry, and that the chief competitor for railway shipping is shipping by truck. Both of these are true claims, but neither is the conclusion.

We can confirm this because neither is supported in any way. For (E), what if I believed that the chief competitor of railroads is cargo donkeys? The author does not try to convince me that I am wrong. (E) is just a take it or leave it, a premise. Conclusions are not take it or leave it. They give us reasons why we should believe.

The same goes for (C). Maybe I believe that there will be a reduction in operating costs for the trucking industry. Have you not heard of Tesla trucks? They are supposed to be way cheaper. Again, the author would not try to convince me otherwise, and that is not the characteristic of a conclusion.

Answer Choice (D) says operating costs for railroads have been reduced as a result of increased reliance on automation. This is just the argument that produces the sub-conclusion/major premise, which then in turn supports the main conclusion.


Comment on this

This is a Most Strongly Supported question.

The stimulus says that in the past, infants who were not breast-fed were fed cow's milk. Then doctors began advising that cow's milk fed to infants should be boiled, as the boiling would sterilize the milk and prevent gastrointestinal infections potentially fatal to infants. And once this advice was widely implemented, there was an alarming increase among infants in the incidence of scurvy, which is caused by vitamin C deficiency. Breast-fed infants, however, did not contract scurvy.

While this is not an ideal experiment, it's what's known as a natural experiment. There are two groups, the "intervention" and "control" groups. The intervention group (boiling cow's milk) exhibited more incidences of scurvy than the control group (breast-fed milk). We suspect that the explanation is that boiling cow's milk was the cause. But how? Why would boiling cow's milk cause the infants to contract scurvy? The stimulus contains another fact: scurvy is caused by vitamin C deficiency. That's a clue about the causal mechanism. Together, the facts (the phenomena) strongly support the hypothesis that boiling cow's milk destroyed vitamin C, which in turn caused scurvy.

Correct Answer Choice (A) says boiled cow's milk makes less vitamin C available to the infant than does the same amount of mother's milk. This is a version of the hypothesis above. That's why it's the right answer. (A) is by no means a “must be true,” but the standard of proof is lower for MSS. You never really reach 100% validity in scientific reasoning anyway. That only happens in formal reasoning.

And notice how similar this is to Resolve Reconcile Explain. (A) would still be correct if this was an RRE question since it would explain the phenomenon in the stimulus. This reveals that the question stem is more or less superficial, and that there is an underlying unity to Logical Reasoning. And here the logic is scientific reasoning. You are presented with a natural experiment that resembles the ideal experiment and asked to come up with a reasonable hypothesis.

What's a natural experiment and why do I put quotations around "control group"? Because natural experiments are less reliable than ideal experiments. That doesn't mean they're not reliable at all. Far from it. Natural experiments can sometimes provide very strong evidence. But they are not ideal because the "control group" didn't control for everything.

Imagine you had an alternative hypothesis, say, the citrus shortage hypothesis. Under that hypothesis, there was a shortage in citrus fruits that just happened to coincide with the intervention (boiling cow's milk). Perhaps it's actually the citrus shortage that caused the increase in scurvy.

Okay, but we can preclude this hypothesis with an ideal experiment. We can run a control group. Hold everything else equal (including access to citrus) and feed the control group breast milk. If they don't develop scurvy, then it can't be the citrus shortage that caused the scurvy. That's kind of like what happened in the natural experiment of the stimulus. The difference is that we have no assurances that the breast-fed babies' exposure to citrus was the same as the intervention babies' exposure. It's possible that breast-fed infants somehow had priority access to oranges. It's just very unlikely. That distance is there, though, in the natural experiment, whereas in the ideal experiment, we have assurances that the control group did in fact control for everything. That's why natural experiments are weaker than ideal ones. How much weaker? That depends in turn on which hypotheses you're trying to eliminate. For the citrus shortage hypothesis, it's highly unlikely that the breast-fed group would have had priority access and, therefore, the natural experiment is not that much weaker than the ideal experiment.

Answer Choice (B) says infants who consume cow's milk that has not been boiled frequently contract potentially fatal gastrointestinal infections. (B) could have been correct if it had said sometimes, potentially, or even in danger of contracting. Doctors recommended the intervention precisely to prevent gastrointestinal infections, so it is a reasonable assumption that they thought of unboiled cow’s milk as an actual risk. But “frequently” is too strong to be supported.

Answer Choice (C) says mother's milk can cause gastrointestinal infections in infants. This is just a mishmash. We do know that unboiled cow's milk potentially can cause gastrointestinal infections, but can mother's milk cause it? Nothing in the stimulus suggests the answer is one way or another.

Answer Choice (D) is really attractive. If you picked (D), you probably read some alternate version of (D) that resembles the following: when doctors advised that cow's milk fed to infants be boiled, they did not know that this intervention would lead to vitamin C deficiency, which then would lead to scurvy.

Then (D) would be pretty well supported. It seems clear that when doctors began advising to boil the milk, they did not anticipate that scurvy would be a consequence. I am guessing they also did not know that infants depended on milk for their vitamin C, because if they had some other source, for example, like orange juice, who cares about the vitamin C in the milk? Just drink your orange juice.

But that is not what (D) says. (D) just says that when doctors advised cow's milk to be boiled, the cause of scurvy was a mystery. This I am not so sure about. It is totally possible that doctors knew the cause of scurvy. Lots of sailors knew vitamin C deficiency was the cause of scurvy for hundreds of years. They just did not think this intervention (boiling cow's milk) would result in vitamin C deficiency and hence scurvy.

Answer Choice (E) says that when doctors advised that cow's milk fed to infants be boiled, most mothers did not breast-feed their infants. We know that when this intervention happened, there were some infants who were drinking cow's milk and some who were breast-fed. But nothing in the stimulus says which set is larger, so (E) is totally unsupported.


7 comments

This is a Parallel Method of Reasoning question.

The stimulus says the railway authority inspector who recently thoroughly checked the tracks testified that they were in good condition. The next claim says, “Thus, since” which means we’re about to get a premise followed by a conclusion. Since the inspector has no bias in the matter (premise), we should thus be suspicious of the newspaper reporter's claims that the tracks are in poor condition (conclusion).

Two people disagree about the condition of the railroad tracks. The argument is that we should believe the inspector, not the reporter. Why? Because the inspector has no bias and said the tracks were in good condition after having thoroughly checked them. The reasoning here is an appeal to authority. Let's now look at the answers and try to match the reasoning above.

Answer Choice (A) says my pottery instructor says that making pottery will not cause repetitive-motion injuries if it is done properly. So far, there seems to be a parallel between the pottery instructor and the inspector. Now we have to find somebody who disagrees and then conclude that we should believe the pottery instructor. But then (A) instead says, “I will probably not get such injuries, for whenever I do pottery, I use the proper techniques that my instructor taught me.” While this argument is fine, it is not analogous to the argument in the stimulus.

Answer Choice (B) says Gardner, a noted paleontologist who has no vested interest in the case, assures us that the alleged dinosaur bones are not old enough to be dinosaurs. Gardner, a relevant expert, is analogous to the inspector. Gardner also has no vested interest in the case, so he has no bias.

(B) goes on to say that we should be skeptical of Penwick's claims to have found dinosaur bones, for Gardner inspected the bones carefully. Gardner’s careful inspection is analogous to the inspector’s thorough check, and Penwick is analogous to the newspaper reporter. And the conclusion is that we should be skeptical of Penwick, which is similar to how we should be suspicious of the reporter. (B) matches the stimulus extremely well. The reasoning here is also one of an appeal to authority.

Answer Choice (C) says the engineer hired by the company that maintains the bridge has examined the bridge and declared it safe. The engineer is a relevant expert who has examined the bridge and declared it safe. While we should be wary of subject matter similarity since LSAT writers like to use that as a trap, so far, (C) is analogous to the stimulus.

But (C) then says that the engineer is the only one to have checked the bridge. This idea is not present in the stimulus. In addition, the disagreement component is missing in (C) too. There is no analogue to the newspaper reporter.

Answer Choice (D) says the reporter who recently interviewed the prime minister said that the prime minister appeared to be in poor health. The paper has opposition leanings, so that is a bias. (D) is already diverging from the stimulus.

If we were to keep the subject matter of (D) but apply the reasoning in the stimulus, we would have to say that the reporter did a careful, thorough check (ideally, we’d change the reporter to a doctor to create the appeal to authority). And since the reporter is from an unbiased newspaper, we should be suspicious of, for example, the prime minister's press secretary, who claims that the prime minister is in good health. The edits above would make (D) better, but (D) does not resemble the stimulus in its current form.

Answer Choice (E) says the snowblower salesperson claims that there will be above-average snowfall this winter, but because the salesperson is biased, we can discount the claim. Since the salesperson is biased, (E) is already ruled out.

If we were to keep the subject matter of (E) but apply the reasoning in the stimulus as we did for (D), we would have to say that the snowblower salesperson recently and thoroughly inspected my snowblower and said that it was in good condition. The salesperson is not biased. Therefore, I should be suspicious of my neighbor's claim that my snowblower is in poor condition. You wouldn’t have the snowblower salesperson offer his opinion on the weather since that’s not within his domain of expertise.


4 comments